Prove the alternating sum of a decreasing sequence converging to 0 is Cauchy.












3












$begingroup$


Let $(x_n)$ be a decreasing sequence with $x_n > 0$ for all $n in mathbb{N}$, and $(x_n) to 0$. Let $(y_n)$ be defined for all $n in mathbb{N}$ by
$$y_n = x_0 - x_1 + x_2 - cdots + (-1)^n x_n .$$



I want to show, using the $varepsilon$ definition, that $(y_n)$ is Cauchy.



I am trying to find, given $varepsilon > 0$, a real number $N$ such that for all $m$ and $n$ with $m > n > N$, $|y_m - y_n| < varepsilon$.



I have been going backwards to try and find $N$, and have
begin{align*}
|y_m - y_n| & = left| (x_0 - x_1 + cdots pm x_m) - (x_0 - x_1 + cdots pm x_n) right| \
|y_m - y_n| & = left| x_{n + 1} - x_{n + 2} + cdots pm x_{m} right| \
|y_m - y_n| & leq | x_{n + 1} | + | x_{n + 2} | + cdots + | x_{m} | \
|y_m - y_n| & leq ?
end{align*}



I do not know how to get a solution from there, and am not sure about the process, particurlary the last step since I feel getting rid of the minuses might prevent me from finding a solution.










share|cite|improve this question









$endgroup$








  • 1




    $begingroup$
    Because the series is alternating and decreasing, I think you can prove by induction on $m$ that $|y_m-y_n| leq |y_n|$.
    $endgroup$
    – Robert Shore
    5 hours ago










  • $begingroup$
    @RobertShore is my answer okay?
    $endgroup$
    – Subhasis Biswas
    5 hours ago










  • $begingroup$
    @RobertShore yes I can definitely show that, but it brings me to the same issue with $|y_m| leq |x_0 - x_1 + cdots pm x_m|$, and I am unsure how to proceed from there.
    $endgroup$
    – oranji
    2 hours ago












  • $begingroup$
    I meant to say you can prove by induction that $|y_m-y_n| leq |x_n|$. Since $lim x_n=0$, choose $N$ such that $n gt N Rightarrow |x_n| lt epsilon$. Then $|y_m-y_n| leq |x_n| lt epsilon$ so ${y_n}$ is Cauchy.
    $endgroup$
    – Robert Shore
    53 mins ago


















3












$begingroup$


Let $(x_n)$ be a decreasing sequence with $x_n > 0$ for all $n in mathbb{N}$, and $(x_n) to 0$. Let $(y_n)$ be defined for all $n in mathbb{N}$ by
$$y_n = x_0 - x_1 + x_2 - cdots + (-1)^n x_n .$$



I want to show, using the $varepsilon$ definition, that $(y_n)$ is Cauchy.



I am trying to find, given $varepsilon > 0$, a real number $N$ such that for all $m$ and $n$ with $m > n > N$, $|y_m - y_n| < varepsilon$.



I have been going backwards to try and find $N$, and have
begin{align*}
|y_m - y_n| & = left| (x_0 - x_1 + cdots pm x_m) - (x_0 - x_1 + cdots pm x_n) right| \
|y_m - y_n| & = left| x_{n + 1} - x_{n + 2} + cdots pm x_{m} right| \
|y_m - y_n| & leq | x_{n + 1} | + | x_{n + 2} | + cdots + | x_{m} | \
|y_m - y_n| & leq ?
end{align*}



I do not know how to get a solution from there, and am not sure about the process, particurlary the last step since I feel getting rid of the minuses might prevent me from finding a solution.










share|cite|improve this question









$endgroup$








  • 1




    $begingroup$
    Because the series is alternating and decreasing, I think you can prove by induction on $m$ that $|y_m-y_n| leq |y_n|$.
    $endgroup$
    – Robert Shore
    5 hours ago










  • $begingroup$
    @RobertShore is my answer okay?
    $endgroup$
    – Subhasis Biswas
    5 hours ago










  • $begingroup$
    @RobertShore yes I can definitely show that, but it brings me to the same issue with $|y_m| leq |x_0 - x_1 + cdots pm x_m|$, and I am unsure how to proceed from there.
    $endgroup$
    – oranji
    2 hours ago












  • $begingroup$
    I meant to say you can prove by induction that $|y_m-y_n| leq |x_n|$. Since $lim x_n=0$, choose $N$ such that $n gt N Rightarrow |x_n| lt epsilon$. Then $|y_m-y_n| leq |x_n| lt epsilon$ so ${y_n}$ is Cauchy.
    $endgroup$
    – Robert Shore
    53 mins ago
















3












3








3


1



$begingroup$


Let $(x_n)$ be a decreasing sequence with $x_n > 0$ for all $n in mathbb{N}$, and $(x_n) to 0$. Let $(y_n)$ be defined for all $n in mathbb{N}$ by
$$y_n = x_0 - x_1 + x_2 - cdots + (-1)^n x_n .$$



I want to show, using the $varepsilon$ definition, that $(y_n)$ is Cauchy.



I am trying to find, given $varepsilon > 0$, a real number $N$ such that for all $m$ and $n$ with $m > n > N$, $|y_m - y_n| < varepsilon$.



I have been going backwards to try and find $N$, and have
begin{align*}
|y_m - y_n| & = left| (x_0 - x_1 + cdots pm x_m) - (x_0 - x_1 + cdots pm x_n) right| \
|y_m - y_n| & = left| x_{n + 1} - x_{n + 2} + cdots pm x_{m} right| \
|y_m - y_n| & leq | x_{n + 1} | + | x_{n + 2} | + cdots + | x_{m} | \
|y_m - y_n| & leq ?
end{align*}



I do not know how to get a solution from there, and am not sure about the process, particurlary the last step since I feel getting rid of the minuses might prevent me from finding a solution.










share|cite|improve this question









$endgroup$




Let $(x_n)$ be a decreasing sequence with $x_n > 0$ for all $n in mathbb{N}$, and $(x_n) to 0$. Let $(y_n)$ be defined for all $n in mathbb{N}$ by
$$y_n = x_0 - x_1 + x_2 - cdots + (-1)^n x_n .$$



I want to show, using the $varepsilon$ definition, that $(y_n)$ is Cauchy.



I am trying to find, given $varepsilon > 0$, a real number $N$ such that for all $m$ and $n$ with $m > n > N$, $|y_m - y_n| < varepsilon$.



I have been going backwards to try and find $N$, and have
begin{align*}
|y_m - y_n| & = left| (x_0 - x_1 + cdots pm x_m) - (x_0 - x_1 + cdots pm x_n) right| \
|y_m - y_n| & = left| x_{n + 1} - x_{n + 2} + cdots pm x_{m} right| \
|y_m - y_n| & leq | x_{n + 1} | + | x_{n + 2} | + cdots + | x_{m} | \
|y_m - y_n| & leq ?
end{align*}



I do not know how to get a solution from there, and am not sure about the process, particurlary the last step since I feel getting rid of the minuses might prevent me from finding a solution.







real-analysis cauchy-sequences






share|cite|improve this question













share|cite|improve this question











share|cite|improve this question




share|cite|improve this question










asked 5 hours ago









oranjioranji

666




666








  • 1




    $begingroup$
    Because the series is alternating and decreasing, I think you can prove by induction on $m$ that $|y_m-y_n| leq |y_n|$.
    $endgroup$
    – Robert Shore
    5 hours ago










  • $begingroup$
    @RobertShore is my answer okay?
    $endgroup$
    – Subhasis Biswas
    5 hours ago










  • $begingroup$
    @RobertShore yes I can definitely show that, but it brings me to the same issue with $|y_m| leq |x_0 - x_1 + cdots pm x_m|$, and I am unsure how to proceed from there.
    $endgroup$
    – oranji
    2 hours ago












  • $begingroup$
    I meant to say you can prove by induction that $|y_m-y_n| leq |x_n|$. Since $lim x_n=0$, choose $N$ such that $n gt N Rightarrow |x_n| lt epsilon$. Then $|y_m-y_n| leq |x_n| lt epsilon$ so ${y_n}$ is Cauchy.
    $endgroup$
    – Robert Shore
    53 mins ago
















  • 1




    $begingroup$
    Because the series is alternating and decreasing, I think you can prove by induction on $m$ that $|y_m-y_n| leq |y_n|$.
    $endgroup$
    – Robert Shore
    5 hours ago










  • $begingroup$
    @RobertShore is my answer okay?
    $endgroup$
    – Subhasis Biswas
    5 hours ago










  • $begingroup$
    @RobertShore yes I can definitely show that, but it brings me to the same issue with $|y_m| leq |x_0 - x_1 + cdots pm x_m|$, and I am unsure how to proceed from there.
    $endgroup$
    – oranji
    2 hours ago












  • $begingroup$
    I meant to say you can prove by induction that $|y_m-y_n| leq |x_n|$. Since $lim x_n=0$, choose $N$ such that $n gt N Rightarrow |x_n| lt epsilon$. Then $|y_m-y_n| leq |x_n| lt epsilon$ so ${y_n}$ is Cauchy.
    $endgroup$
    – Robert Shore
    53 mins ago










1




1




$begingroup$
Because the series is alternating and decreasing, I think you can prove by induction on $m$ that $|y_m-y_n| leq |y_n|$.
$endgroup$
– Robert Shore
5 hours ago




$begingroup$
Because the series is alternating and decreasing, I think you can prove by induction on $m$ that $|y_m-y_n| leq |y_n|$.
$endgroup$
– Robert Shore
5 hours ago












$begingroup$
@RobertShore is my answer okay?
$endgroup$
– Subhasis Biswas
5 hours ago




$begingroup$
@RobertShore is my answer okay?
$endgroup$
– Subhasis Biswas
5 hours ago












$begingroup$
@RobertShore yes I can definitely show that, but it brings me to the same issue with $|y_m| leq |x_0 - x_1 + cdots pm x_m|$, and I am unsure how to proceed from there.
$endgroup$
– oranji
2 hours ago






$begingroup$
@RobertShore yes I can definitely show that, but it brings me to the same issue with $|y_m| leq |x_0 - x_1 + cdots pm x_m|$, and I am unsure how to proceed from there.
$endgroup$
– oranji
2 hours ago














$begingroup$
I meant to say you can prove by induction that $|y_m-y_n| leq |x_n|$. Since $lim x_n=0$, choose $N$ such that $n gt N Rightarrow |x_n| lt epsilon$. Then $|y_m-y_n| leq |x_n| lt epsilon$ so ${y_n}$ is Cauchy.
$endgroup$
– Robert Shore
53 mins ago






$begingroup$
I meant to say you can prove by induction that $|y_m-y_n| leq |x_n|$. Since $lim x_n=0$, choose $N$ such that $n gt N Rightarrow |x_n| lt epsilon$. Then $|y_m-y_n| leq |x_n| lt epsilon$ so ${y_n}$ is Cauchy.
$endgroup$
– Robert Shore
53 mins ago












2 Answers
2






active

oldest

votes


















3












$begingroup$

To see that the sequence of partial sums is Cauchy, you cannot use the triangle inequality directly as you did. A famous counter example here is $sum_{k=1}^{infty}frac{(-1)^k}{k}$.



What you can do is grouping the terms of the partial sums $s_n= sum_{j=1}^n(-1)^jx_j$ as follows:




  • Let $m = n+k, k,n in mathbb{N}$


Now, you can write $|s_{m} - s_n|$ in two different ways:



$$|s_{n+k} - s_n| = begin{cases}
|x_{n+1} - (x_{n+2}-x_{n+3}) - cdots - (x_{n+2i}-x_{n+2i+1})| & k = 2i+1 \
|x_{n+1} - (x_{n+2}-x_{n+3}) - cdots - (x_{n+2i-2}-x_{n+2i-1}) - x_{2i}| & k = 2i \
end{cases}
$$



$$|s_{n+k} - s_n| = begin{cases}
|(x_{n+1} - x_{n+2}) + cdots + (x_{n+2i-1}-x_{n+2i}) + x_{n+2i+1}| & k = 2i+1 \
|(x_{n+1} - x_{n+2}) + cdots + (x_{n+2i-1}-x_{n+2i}) | & k = 2i \
end{cases}
$$



Using the fact that $x_n searrow 0$, it follows immediately that for all $k in mathbb{N}$ holds
$$|s_{n+k} - s_n| leq x_{n+1}$$



Hence, for $epsilon > 0$ choose $N_{epsilon}$ such that $x_{N_{epsilon}} < epsilon$. Then, for all $m> n > N_{epsilon}$ you have
$$|s_{m} - s_n| leq x_{n+1} leq x_{N_{epsilon}} < epsilon$$






share|cite|improve this answer











$endgroup$













  • $begingroup$
    This is exactly what I was about to do.
    $endgroup$
    – Subhasis Biswas
    2 hours ago






  • 1




    $begingroup$
    @SubhasisBiswas So, I did it for you :-D
    $endgroup$
    – trancelocation
    2 hours ago



















4












$begingroup$

This is also known as the "Leibnitz's Test".



We write $s_n = x_1-x_2+x_3-...+(-1)^{n+1}x_n$



$s_{2n+2}-s_{2n}=u_{2n+1}-u_{2n+2} geq0$ for all $n$.



$s_{2n+1}-s_{2n-1}=-u_{2n}+u_{2n+1} leq 0$



$s_{2n} =u_1 -(u_2-u_3)-(u_4-u_5)...-u_{2n} leq u_1$, i.e. a monotone increasing sequence bounded above.



$s_{2n+1} =(u_1 -u_2)+(u_3-u_4)+...+u_{2n+1} geq u_1-u_2$, i.e. a monotone decreasing sequence bounded below.



Hence, both are convergent subsequences of $(s_n)$. But, we have $lim (s_{2n+1}-s_{2n})=u_{2n+1}=0$, therefore, they converge to the same limit.



Hence, $(s_n)$ converges, i.e. it is Cauchy.



Note: We conclude that $(s_n)$ converges because the indices of the two subsequences $(s_{2n})$ and $(s_{2n+1})$ i.e. $U ={ 2n+1 : n in mathbb{N}}$ and $V ={ 2n : n in mathbb{N}}$ form a partition of $mathbb{N}$ and they both converge to the same limit.






share|cite|improve this answer









$endgroup$













  • $begingroup$
    I want to use the $varepsilon$ definition of a Cauchy sequence, and not the fact that all convergent sequences are Cauchy, which is why I cannot use this solution.
    $endgroup$
    – oranji
    2 hours ago










  • $begingroup$
    I'll edit this answer.
    $endgroup$
    – Subhasis Biswas
    2 hours ago












Your Answer








StackExchange.ready(function() {
var channelOptions = {
tags: "".split(" "),
id: "69"
};
initTagRenderer("".split(" "), "".split(" "), channelOptions);

StackExchange.using("externalEditor", function() {
// Have to fire editor after snippets, if snippets enabled
if (StackExchange.settings.snippets.snippetsEnabled) {
StackExchange.using("snippets", function() {
createEditor();
});
}
else {
createEditor();
}
});

function createEditor() {
StackExchange.prepareEditor({
heartbeatType: 'answer',
autoActivateHeartbeat: false,
convertImagesToLinks: true,
noModals: true,
showLowRepImageUploadWarning: true,
reputationToPostImages: 10,
bindNavPrevention: true,
postfix: "",
imageUploader: {
brandingHtml: "Powered by u003ca class="icon-imgur-white" href="https://imgur.com/"u003eu003c/au003e",
contentPolicyHtml: "User contributions licensed under u003ca href="https://creativecommons.org/licenses/by-sa/3.0/"u003ecc by-sa 3.0 with attribution requiredu003c/au003e u003ca href="https://stackoverflow.com/legal/content-policy"u003e(content policy)u003c/au003e",
allowUrls: true
},
noCode: true, onDemand: true,
discardSelector: ".discard-answer"
,immediatelyShowMarkdownHelp:true
});


}
});














draft saved

draft discarded


















StackExchange.ready(
function () {
StackExchange.openid.initPostLogin('.new-post-login', 'https%3a%2f%2fmath.stackexchange.com%2fquestions%2f3201256%2fprove-the-alternating-sum-of-a-decreasing-sequence-converging-to-0-is-cauchy%23new-answer', 'question_page');
}
);

Post as a guest















Required, but never shown

























2 Answers
2






active

oldest

votes








2 Answers
2






active

oldest

votes









active

oldest

votes






active

oldest

votes









3












$begingroup$

To see that the sequence of partial sums is Cauchy, you cannot use the triangle inequality directly as you did. A famous counter example here is $sum_{k=1}^{infty}frac{(-1)^k}{k}$.



What you can do is grouping the terms of the partial sums $s_n= sum_{j=1}^n(-1)^jx_j$ as follows:




  • Let $m = n+k, k,n in mathbb{N}$


Now, you can write $|s_{m} - s_n|$ in two different ways:



$$|s_{n+k} - s_n| = begin{cases}
|x_{n+1} - (x_{n+2}-x_{n+3}) - cdots - (x_{n+2i}-x_{n+2i+1})| & k = 2i+1 \
|x_{n+1} - (x_{n+2}-x_{n+3}) - cdots - (x_{n+2i-2}-x_{n+2i-1}) - x_{2i}| & k = 2i \
end{cases}
$$



$$|s_{n+k} - s_n| = begin{cases}
|(x_{n+1} - x_{n+2}) + cdots + (x_{n+2i-1}-x_{n+2i}) + x_{n+2i+1}| & k = 2i+1 \
|(x_{n+1} - x_{n+2}) + cdots + (x_{n+2i-1}-x_{n+2i}) | & k = 2i \
end{cases}
$$



Using the fact that $x_n searrow 0$, it follows immediately that for all $k in mathbb{N}$ holds
$$|s_{n+k} - s_n| leq x_{n+1}$$



Hence, for $epsilon > 0$ choose $N_{epsilon}$ such that $x_{N_{epsilon}} < epsilon$. Then, for all $m> n > N_{epsilon}$ you have
$$|s_{m} - s_n| leq x_{n+1} leq x_{N_{epsilon}} < epsilon$$






share|cite|improve this answer











$endgroup$













  • $begingroup$
    This is exactly what I was about to do.
    $endgroup$
    – Subhasis Biswas
    2 hours ago






  • 1




    $begingroup$
    @SubhasisBiswas So, I did it for you :-D
    $endgroup$
    – trancelocation
    2 hours ago
















3












$begingroup$

To see that the sequence of partial sums is Cauchy, you cannot use the triangle inequality directly as you did. A famous counter example here is $sum_{k=1}^{infty}frac{(-1)^k}{k}$.



What you can do is grouping the terms of the partial sums $s_n= sum_{j=1}^n(-1)^jx_j$ as follows:




  • Let $m = n+k, k,n in mathbb{N}$


Now, you can write $|s_{m} - s_n|$ in two different ways:



$$|s_{n+k} - s_n| = begin{cases}
|x_{n+1} - (x_{n+2}-x_{n+3}) - cdots - (x_{n+2i}-x_{n+2i+1})| & k = 2i+1 \
|x_{n+1} - (x_{n+2}-x_{n+3}) - cdots - (x_{n+2i-2}-x_{n+2i-1}) - x_{2i}| & k = 2i \
end{cases}
$$



$$|s_{n+k} - s_n| = begin{cases}
|(x_{n+1} - x_{n+2}) + cdots + (x_{n+2i-1}-x_{n+2i}) + x_{n+2i+1}| & k = 2i+1 \
|(x_{n+1} - x_{n+2}) + cdots + (x_{n+2i-1}-x_{n+2i}) | & k = 2i \
end{cases}
$$



Using the fact that $x_n searrow 0$, it follows immediately that for all $k in mathbb{N}$ holds
$$|s_{n+k} - s_n| leq x_{n+1}$$



Hence, for $epsilon > 0$ choose $N_{epsilon}$ such that $x_{N_{epsilon}} < epsilon$. Then, for all $m> n > N_{epsilon}$ you have
$$|s_{m} - s_n| leq x_{n+1} leq x_{N_{epsilon}} < epsilon$$






share|cite|improve this answer











$endgroup$













  • $begingroup$
    This is exactly what I was about to do.
    $endgroup$
    – Subhasis Biswas
    2 hours ago






  • 1




    $begingroup$
    @SubhasisBiswas So, I did it for you :-D
    $endgroup$
    – trancelocation
    2 hours ago














3












3








3





$begingroup$

To see that the sequence of partial sums is Cauchy, you cannot use the triangle inequality directly as you did. A famous counter example here is $sum_{k=1}^{infty}frac{(-1)^k}{k}$.



What you can do is grouping the terms of the partial sums $s_n= sum_{j=1}^n(-1)^jx_j$ as follows:




  • Let $m = n+k, k,n in mathbb{N}$


Now, you can write $|s_{m} - s_n|$ in two different ways:



$$|s_{n+k} - s_n| = begin{cases}
|x_{n+1} - (x_{n+2}-x_{n+3}) - cdots - (x_{n+2i}-x_{n+2i+1})| & k = 2i+1 \
|x_{n+1} - (x_{n+2}-x_{n+3}) - cdots - (x_{n+2i-2}-x_{n+2i-1}) - x_{2i}| & k = 2i \
end{cases}
$$



$$|s_{n+k} - s_n| = begin{cases}
|(x_{n+1} - x_{n+2}) + cdots + (x_{n+2i-1}-x_{n+2i}) + x_{n+2i+1}| & k = 2i+1 \
|(x_{n+1} - x_{n+2}) + cdots + (x_{n+2i-1}-x_{n+2i}) | & k = 2i \
end{cases}
$$



Using the fact that $x_n searrow 0$, it follows immediately that for all $k in mathbb{N}$ holds
$$|s_{n+k} - s_n| leq x_{n+1}$$



Hence, for $epsilon > 0$ choose $N_{epsilon}$ such that $x_{N_{epsilon}} < epsilon$. Then, for all $m> n > N_{epsilon}$ you have
$$|s_{m} - s_n| leq x_{n+1} leq x_{N_{epsilon}} < epsilon$$






share|cite|improve this answer











$endgroup$



To see that the sequence of partial sums is Cauchy, you cannot use the triangle inequality directly as you did. A famous counter example here is $sum_{k=1}^{infty}frac{(-1)^k}{k}$.



What you can do is grouping the terms of the partial sums $s_n= sum_{j=1}^n(-1)^jx_j$ as follows:




  • Let $m = n+k, k,n in mathbb{N}$


Now, you can write $|s_{m} - s_n|$ in two different ways:



$$|s_{n+k} - s_n| = begin{cases}
|x_{n+1} - (x_{n+2}-x_{n+3}) - cdots - (x_{n+2i}-x_{n+2i+1})| & k = 2i+1 \
|x_{n+1} - (x_{n+2}-x_{n+3}) - cdots - (x_{n+2i-2}-x_{n+2i-1}) - x_{2i}| & k = 2i \
end{cases}
$$



$$|s_{n+k} - s_n| = begin{cases}
|(x_{n+1} - x_{n+2}) + cdots + (x_{n+2i-1}-x_{n+2i}) + x_{n+2i+1}| & k = 2i+1 \
|(x_{n+1} - x_{n+2}) + cdots + (x_{n+2i-1}-x_{n+2i}) | & k = 2i \
end{cases}
$$



Using the fact that $x_n searrow 0$, it follows immediately that for all $k in mathbb{N}$ holds
$$|s_{n+k} - s_n| leq x_{n+1}$$



Hence, for $epsilon > 0$ choose $N_{epsilon}$ such that $x_{N_{epsilon}} < epsilon$. Then, for all $m> n > N_{epsilon}$ you have
$$|s_{m} - s_n| leq x_{n+1} leq x_{N_{epsilon}} < epsilon$$







share|cite|improve this answer














share|cite|improve this answer



share|cite|improve this answer








edited 2 hours ago

























answered 2 hours ago









trancelocationtrancelocation

14.6k1929




14.6k1929












  • $begingroup$
    This is exactly what I was about to do.
    $endgroup$
    – Subhasis Biswas
    2 hours ago






  • 1




    $begingroup$
    @SubhasisBiswas So, I did it for you :-D
    $endgroup$
    – trancelocation
    2 hours ago


















  • $begingroup$
    This is exactly what I was about to do.
    $endgroup$
    – Subhasis Biswas
    2 hours ago






  • 1




    $begingroup$
    @SubhasisBiswas So, I did it for you :-D
    $endgroup$
    – trancelocation
    2 hours ago
















$begingroup$
This is exactly what I was about to do.
$endgroup$
– Subhasis Biswas
2 hours ago




$begingroup$
This is exactly what I was about to do.
$endgroup$
– Subhasis Biswas
2 hours ago




1




1




$begingroup$
@SubhasisBiswas So, I did it for you :-D
$endgroup$
– trancelocation
2 hours ago




$begingroup$
@SubhasisBiswas So, I did it for you :-D
$endgroup$
– trancelocation
2 hours ago











4












$begingroup$

This is also known as the "Leibnitz's Test".



We write $s_n = x_1-x_2+x_3-...+(-1)^{n+1}x_n$



$s_{2n+2}-s_{2n}=u_{2n+1}-u_{2n+2} geq0$ for all $n$.



$s_{2n+1}-s_{2n-1}=-u_{2n}+u_{2n+1} leq 0$



$s_{2n} =u_1 -(u_2-u_3)-(u_4-u_5)...-u_{2n} leq u_1$, i.e. a monotone increasing sequence bounded above.



$s_{2n+1} =(u_1 -u_2)+(u_3-u_4)+...+u_{2n+1} geq u_1-u_2$, i.e. a monotone decreasing sequence bounded below.



Hence, both are convergent subsequences of $(s_n)$. But, we have $lim (s_{2n+1}-s_{2n})=u_{2n+1}=0$, therefore, they converge to the same limit.



Hence, $(s_n)$ converges, i.e. it is Cauchy.



Note: We conclude that $(s_n)$ converges because the indices of the two subsequences $(s_{2n})$ and $(s_{2n+1})$ i.e. $U ={ 2n+1 : n in mathbb{N}}$ and $V ={ 2n : n in mathbb{N}}$ form a partition of $mathbb{N}$ and they both converge to the same limit.






share|cite|improve this answer









$endgroup$













  • $begingroup$
    I want to use the $varepsilon$ definition of a Cauchy sequence, and not the fact that all convergent sequences are Cauchy, which is why I cannot use this solution.
    $endgroup$
    – oranji
    2 hours ago










  • $begingroup$
    I'll edit this answer.
    $endgroup$
    – Subhasis Biswas
    2 hours ago
















4












$begingroup$

This is also known as the "Leibnitz's Test".



We write $s_n = x_1-x_2+x_3-...+(-1)^{n+1}x_n$



$s_{2n+2}-s_{2n}=u_{2n+1}-u_{2n+2} geq0$ for all $n$.



$s_{2n+1}-s_{2n-1}=-u_{2n}+u_{2n+1} leq 0$



$s_{2n} =u_1 -(u_2-u_3)-(u_4-u_5)...-u_{2n} leq u_1$, i.e. a monotone increasing sequence bounded above.



$s_{2n+1} =(u_1 -u_2)+(u_3-u_4)+...+u_{2n+1} geq u_1-u_2$, i.e. a monotone decreasing sequence bounded below.



Hence, both are convergent subsequences of $(s_n)$. But, we have $lim (s_{2n+1}-s_{2n})=u_{2n+1}=0$, therefore, they converge to the same limit.



Hence, $(s_n)$ converges, i.e. it is Cauchy.



Note: We conclude that $(s_n)$ converges because the indices of the two subsequences $(s_{2n})$ and $(s_{2n+1})$ i.e. $U ={ 2n+1 : n in mathbb{N}}$ and $V ={ 2n : n in mathbb{N}}$ form a partition of $mathbb{N}$ and they both converge to the same limit.






share|cite|improve this answer









$endgroup$













  • $begingroup$
    I want to use the $varepsilon$ definition of a Cauchy sequence, and not the fact that all convergent sequences are Cauchy, which is why I cannot use this solution.
    $endgroup$
    – oranji
    2 hours ago










  • $begingroup$
    I'll edit this answer.
    $endgroup$
    – Subhasis Biswas
    2 hours ago














4












4








4





$begingroup$

This is also known as the "Leibnitz's Test".



We write $s_n = x_1-x_2+x_3-...+(-1)^{n+1}x_n$



$s_{2n+2}-s_{2n}=u_{2n+1}-u_{2n+2} geq0$ for all $n$.



$s_{2n+1}-s_{2n-1}=-u_{2n}+u_{2n+1} leq 0$



$s_{2n} =u_1 -(u_2-u_3)-(u_4-u_5)...-u_{2n} leq u_1$, i.e. a monotone increasing sequence bounded above.



$s_{2n+1} =(u_1 -u_2)+(u_3-u_4)+...+u_{2n+1} geq u_1-u_2$, i.e. a monotone decreasing sequence bounded below.



Hence, both are convergent subsequences of $(s_n)$. But, we have $lim (s_{2n+1}-s_{2n})=u_{2n+1}=0$, therefore, they converge to the same limit.



Hence, $(s_n)$ converges, i.e. it is Cauchy.



Note: We conclude that $(s_n)$ converges because the indices of the two subsequences $(s_{2n})$ and $(s_{2n+1})$ i.e. $U ={ 2n+1 : n in mathbb{N}}$ and $V ={ 2n : n in mathbb{N}}$ form a partition of $mathbb{N}$ and they both converge to the same limit.






share|cite|improve this answer









$endgroup$



This is also known as the "Leibnitz's Test".



We write $s_n = x_1-x_2+x_3-...+(-1)^{n+1}x_n$



$s_{2n+2}-s_{2n}=u_{2n+1}-u_{2n+2} geq0$ for all $n$.



$s_{2n+1}-s_{2n-1}=-u_{2n}+u_{2n+1} leq 0$



$s_{2n} =u_1 -(u_2-u_3)-(u_4-u_5)...-u_{2n} leq u_1$, i.e. a monotone increasing sequence bounded above.



$s_{2n+1} =(u_1 -u_2)+(u_3-u_4)+...+u_{2n+1} geq u_1-u_2$, i.e. a monotone decreasing sequence bounded below.



Hence, both are convergent subsequences of $(s_n)$. But, we have $lim (s_{2n+1}-s_{2n})=u_{2n+1}=0$, therefore, they converge to the same limit.



Hence, $(s_n)$ converges, i.e. it is Cauchy.



Note: We conclude that $(s_n)$ converges because the indices of the two subsequences $(s_{2n})$ and $(s_{2n+1})$ i.e. $U ={ 2n+1 : n in mathbb{N}}$ and $V ={ 2n : n in mathbb{N}}$ form a partition of $mathbb{N}$ and they both converge to the same limit.







share|cite|improve this answer












share|cite|improve this answer



share|cite|improve this answer










answered 5 hours ago









Subhasis BiswasSubhasis Biswas

628512




628512












  • $begingroup$
    I want to use the $varepsilon$ definition of a Cauchy sequence, and not the fact that all convergent sequences are Cauchy, which is why I cannot use this solution.
    $endgroup$
    – oranji
    2 hours ago










  • $begingroup$
    I'll edit this answer.
    $endgroup$
    – Subhasis Biswas
    2 hours ago


















  • $begingroup$
    I want to use the $varepsilon$ definition of a Cauchy sequence, and not the fact that all convergent sequences are Cauchy, which is why I cannot use this solution.
    $endgroup$
    – oranji
    2 hours ago










  • $begingroup$
    I'll edit this answer.
    $endgroup$
    – Subhasis Biswas
    2 hours ago
















$begingroup$
I want to use the $varepsilon$ definition of a Cauchy sequence, and not the fact that all convergent sequences are Cauchy, which is why I cannot use this solution.
$endgroup$
– oranji
2 hours ago




$begingroup$
I want to use the $varepsilon$ definition of a Cauchy sequence, and not the fact that all convergent sequences are Cauchy, which is why I cannot use this solution.
$endgroup$
– oranji
2 hours ago












$begingroup$
I'll edit this answer.
$endgroup$
– Subhasis Biswas
2 hours ago




$begingroup$
I'll edit this answer.
$endgroup$
– Subhasis Biswas
2 hours ago


















draft saved

draft discarded




















































Thanks for contributing an answer to Mathematics Stack Exchange!


  • Please be sure to answer the question. Provide details and share your research!

But avoid



  • Asking for help, clarification, or responding to other answers.

  • Making statements based on opinion; back them up with references or personal experience.


Use MathJax to format equations. MathJax reference.


To learn more, see our tips on writing great answers.




draft saved


draft discarded














StackExchange.ready(
function () {
StackExchange.openid.initPostLogin('.new-post-login', 'https%3a%2f%2fmath.stackexchange.com%2fquestions%2f3201256%2fprove-the-alternating-sum-of-a-decreasing-sequence-converging-to-0-is-cauchy%23new-answer', 'question_page');
}
);

Post as a guest















Required, but never shown





















































Required, but never shown














Required, but never shown












Required, but never shown







Required, but never shown

































Required, but never shown














Required, but never shown












Required, but never shown







Required, but never shown







Popular posts from this blog

How did Captain America manage to do this?

迪纳利

南乌拉尔铁路局